LSAT and Law School Admissions Forum

Get expert LSAT preparation and law school admissions advice from PowerScore Test Preparation.

 Administrator
PowerScore Staff
  • PowerScore Staff
  • Posts: 8917
  • Joined: Feb 02, 2011
|
#24969
Complete Question Explanation

Strengthen—PR. The correct answer choice is (D)

This argument by analogy concludes that the government would not be justified in suppressing the view that people should eat a diet that includes uncooked meat. The author provides an analogy stating that since the government would not be justified in suppressing potentially harmful political speech, it is not justified in suppressing this view on nutrition.

The phrasing of the question stem is particularly difficult for students. It uses the word “justify,” which often makes students think it is a Justify the Conclusion question. However, we are looking for the answer choice that most helps to justify the conclusion, which is actually phrasing for a Strengthen question. A Justify the Conclusion question will not contain limiting words, such as “helps.” Therefore, we know that this cannot be a Justify question, and must be a Strengthen question.

Answer choice (A): This answer choice has no impact on the argument. We do not know what portion of society would think eating uncooked meat is beneficial. Therefore, this answer choice does not directly connect with the facts in the stimulus.

Answer choice (B): Much like answer choice (A), the stimulus doesn’t give us information on what is in the best interest of society. Arguably, they suggest that silencing speech on eating uncooked meat would be in the best interest of society, as eating uncooked meat could be dangerous. Ultimately, since the stimulus did not give us information about what is best for society, this answer choice does not impact the argument.

Answer choice (C): The conclusion in the stimulus is not about what individuals ought to do; it is about what the government ought to do. We are looking for an answer choice that provides a principle that strengthens the author’s opinion about the correct action the government should take.

Answer choice (D): This is the correct answer choice. The conclusion of the argument states that the government should not censor the potentially dangerous speech. This answer choice strengthens the conclusion by stating the government should not silence an opinion just because it could be harmful. Thus, since the possibility of harm was the only reason given for possibly censoring the opinion, this answer choice strengthens the argument.

Answer choice (E): This answer choice, like answer choice (C), addresses what individuals ought to do, not what the government ought to do. Thus, it is incorrect for the same reason.

Get the most out of your LSAT Prep Plus subscription.

Analyze and track your performance with our Testing and Analytics Package.